Derivando la expectativa de [X^,H^][X^,H^][\hat X,\hat H]

Para una partícula libre de masa metro , con hamiltoniano

H ^ = PAG ^ 2 2 metro ,

dónde

PAG ^ = i X .

La relación conmutativa está dada por

(1) [ X ^ , H ^ ] = i metro PAG ^

En el estado propio común de H ^ y PAG ^ , | mi , pag , podemos hacer lo siguiente?

mi , pag | [ X ^ , H ^ ] | mi , pag = mi , pag | X ^ ( H ^ | mi , pag ) ( mi , pag | H ^ ) X ^ | mi , pag = mi , pag | X ^ ( mi | mi , pag ) ( mi , pag | mi ) X ^ | mi , pag = mi ( mi , pag | X ^ | mi , pag mi , pag | X ^ | mi , pag ) = 0

Desde el H ^ es hermítica, la derivación anterior no parece mostrar ningún defecto. Dada la relación conmutativa, ecuación (1), sabemos que el resultado es incorrecto. ¿Qué hay de malo en la derivación anterior?

[ EDITAR ]

Siguiendo el comentario de Luboš Motl, he encontrado la solución y me gustaría compartirla aquí. El enlace proporcionado por Qmechanic tenía la solución estrechamente relacionada con esta pregunta.

mi , pag | [ X ^ , H ^ ] | mi , pag = mi , pag | X ^ ( H ^ | mi , pag ) ( mi , pag | H ^ ) X ^ | mi , pag = ( mi mi ) mi , pag | X ^ | mi , pag

Tenga en cuenta que:

mi mi = pag 2 2 metro pag 2 2 metro = ( pag + pag ) ( pag pag ) 2 metro

mi , pag | X ^ | mi , pag = i d ( pag pag )

dónde d ( ) es la derivada de la función de Dirac, con respecto a pag .

Entonces obtenemos

( mi mi ) mi , pag | X ^ | mi , pag = i ( pag + pag ) 2 metro ( pag pag ) d ( pag pag ) = i ( pag + pag ) 2 metro ( d ( pag pag ) ) = i ( pag + pag ) 2 metro d ( pag pag )

A medida que tomamos el límite pag pag :

yo i metro pag pag i ( pag + pag ) 2 metro d ( pag pag ) i metro pag d ( pag pag )

Es una falla muy sutil, pero ¿ha intentado intercalar al operador entre estados algo más generales? mi , pag | y | mi , pag ?
@LubošMotl en realidad probé el caso general del sándwich, pero lo descarté sin pensarlo. Como lo mencionaste, me di cuenta de que es realmente un buen punto para atacar: yo i metro mi mi ⟨e,p| X ^ ( H ^ |e′,p′⟩) - (⟨e,p| H ^ ) X ^ |e′,p′⟩ = yo i metro mi mi (e' - e) ⟨e,p| X ^ |e′,p′⟩. Desde yo i metro mi mi ( mi mi ) 0 y yo i metro mi mi mi , pag | X ^ | mi , pag , no podemos descartar ninguno de los dos factores. Intentaré resolverlo con más detalle. Te voté.
Hola usuario, sí. Solo para estar seguro, trate de pensar en el valor de la función X d ( X ) en X = 0 .

Respuestas (2)

Como es costumbre en tal pregunta, señalaré esto artículo, que trata de manera excelente los problemas que tiene el formalismo de Dirac.

Ahora, en su ejemplo concreto, el problema radica en los estados de energía/momento | pag 0 mismos, que no son normalizables, ya que la función de onda asociada es la transformada de Fourier de d ( pag pag 0 ) , Lo que significa que ψ | pag 0 ( X ) = mi i X pag 0 . Si ahora tratas de calcular el producto interno, encuentras:

pag 0 | pag 0 = ψ | pag 0 ( X ) ψ ¯ | pag 0 ( X ) d X = mi i X pag 0 mi i X pag 0 d X = 1 d X

Por lo tanto, los estados propios de cantidad de movimiento no son normalizables, y escribir cosas como pag 0 | X | pag 0 pag 0 | X | pag 0 es realmente absurdo, porque estás restando dos infinitos. En particular, no es 0 .

ese parece ser un buen artículo - te voté por eso. Examinaré el documento con más detalles. Pero no estoy satisfecho con su respuesta, que escuché en otro lugar antes. Déjame explicarte por qué. No creo que nadie pueda obtener un resultado en particular, para restar los dos infinitos que mencionaste: < pag 0 | X ^ | pag 0 > < pag 0 | X ^ | pag 0 > . Son literalmente dos entidades idénticas, lo cual fue un rompecabezas desde el principio. Creo que Luboš Motl señaló la solución real: la expectativa no se definió correctamente, excepto de manera limitante.
No creo que estemos en desacuerdo: nadie puede obtener un resultado por la diferencia, porque no tiene sentido, ya que sus componentes no existen. Ciertamente no voy a envidiar que estés más feliz con el enfoque de Luboš para atacar este problema, ya que ciertamente es un poco más al grano :)
Agradezco mucho tu respuesta. Permítanme explicar mi punto un poco más. La motivación de mi pregunta era buscar el consenso entre los dos enfoques de cálculo < pag 0 | [ X ^ , H ^ ] | pag 0 > . La segunda forma de aplicar la propiedad hermitiana de H ^ debe llegar al mismo resultado, en lugar de simplemente indicar su consecuencia "no-go". Por ejemplo, si pudiéramos reclamar el Hermitian-ness de H ^ no es válido en este caso, eso se consideraría un consenso lógico.
Al indicar el ejemplo anterior, realmente no quiero decir H ^ no es hermitiano aquí.
Me temo que "La segunda forma de aplicar la propiedad hermitiana de H ^ debería obtener el mismo resultado" es una expectativa que debe abandonar, pero su primera forma (supongo que quiere decir usar la relación de conmutación) tampoco produce un resultado definitivo: pag 0 | [ X ^ , H ^ ] | pag 0 = i metro 1 pag 0 | PAG ^ | pag 0 = i metro 1 pag 0 pag 0 | pag 0 , donde esta última expresión es manifiestamente infinita. Así que tienes dos enfoques, y ambos no dan ningún resultado concreto. ¿Buscas algo más?
Estoy de acuerdo en que la primera forma (usando la relación de conmutación) da como resultado un infinito ~ pag 0 pag 0 | pag 0 . Pero su significado físico es claro: la expectativa alcanza su punto máximo en pag 0 . La segunda forma no conduce a ningún resultado físico - es que únicamente debido a la aplicación de la hermética-ness de H ^ ? Yo no lo compraría ;). Si se debe a la normalización mal definida, ¿por qué no podría dar lo mismo? d -como resultado? No hay ninguna razón por la que no pueda, si aplicamos el "hermitismo" correctamente.
H ^ es Hermitian (por cierto, la palabra "correcta" para "Hermitian-ness" es Hermiticidad ), no hay duda al respecto. No siempre es autoadjunto, pero ese no es el problema aquí. Después de haber releído el documento que vinculé, creo que el problema podría ser más bien que | pag 0 ni siquiera está en el dominio de definición del operador X ^ , pero necesito algo de tiempo para tratar de resolverlo.
Te voté por señalarme el nombre "Hermiticidad".

Una pregunta difícil, de verdad. Aparte del hecho de que su | mi , pag vector no pertenece a L 2 (por lo tanto, no puede tomar productos escalares de él), no veo ningún otro defecto. Eso, en mi opinión, significa que tienes un buen argumento para probar la siguiente afirmación matemática:

Dejar H sea ​​un espacio de Hilbert separable, 0 z C . No hay operadores autoadjuntos A y B con espectro discreto no vacío diferente de cero tal que [ A , B ] = z .

Estrechamente relacionado con ese hecho, el siguiente resultado de Von Neumann: hasta multiplicidad y equivalencia unitaria, las relaciones [ A , B ] = i (en su forma exponenciada) se realizan únicamente por A = X (operador de multiplicación) y B = i X , que de hecho no tienen espectro discreto.

EDITADO (en respuesta al comentario, también la declaración anterior se ha editado ligeramente, para ser más precisos):

Un número λ R está en el espectro discreto de A ( llamado σ d i s C ( A ) ) si existe al menos una ψ λ H (el espacio de Hilbert, generalmente L 2 ( R d ) ) tal que

A ψ λ = λ ψ λ .
Supongamos que existe A y B autoadjunto tal que 0 λ σ d i s C ( B ) y [ A , B ] = z (en un dominio denso adecuado). Ahora se sigue que (en otro dominio adecuado)
[ A , B 2 ] = 2 z B .
Dejar ψ λ H ser una de las funciones propias de B asociado a λ . Por un lado,
2 z ψ λ , B ψ λ H = 2 z λ ψ λ H 2 ;
en el otro
ψ λ , A B 2 ψ λ H ψ λ , B 2 A ψ λ H = 0
como sugeriste. Eso es absurdo, ya que z , λ y ψ λ H 2 son diferentes de cero.

De ello se deduce que no puede tener dos operadores autoadjuntos tales que [ A , B ] = z y σ d i s C ( B ) { 0 } , . El razonamiento anterior no funciona si no hay función propia ψ λ H (porque con funciones propias formales no está permitido tomar normas o productos escalares: no son finitos).

Disculpe mi experiencia insuficiente en análisis matemático riguroso. Tengo una pregunta sobre lo que realmente quiere decir. ¿Por qué el hecho de que el vector "∣e,p⟩ no pertenezca a L 2 " tiene algo que ver con la declaración: "no hay operadores A y B autoadjuntos con espectro discreto no vacío tal que [A,B]=z"? Más particularmente, ¿cuál es el problema con el espectro discreto no vacío? ¡Gracias!
He editado la respuesta, mi respuesta habría sido demasiado larga para un comentario.
Veo lo que está diciendo, pero me gustaría mencionar algunos puntos: 1) su espectro (en la primera ecuación) debe ser para el operador B, en lugar de A; 2) de su respuesta editada, no parece importar si el espectro es discreto o continuo; 3) para el ejemplo específico en la pregunta, ¿está indicando H ^ es no hermitiano (mi comprensión autoadjunta significa lo mismo)?
Bueno: 1) en la primera ecuación estoy definiendo el significado de espectro discreto, entonces A o B no importa, pero estoy de acuerdo en que puede ser un poco confuso con la notación a continuación; 2) Importa, porque estoy definiendo el espectro discreto de manera que hay al menos una función propia en el espacio de Hilbert (¡no es el caso para todos los valores en el espectro en general!); 3) auto-adjunto significa simétrico (hermitiano) y algo más (su dominio es igual al dominio del adjunto) ¡hay que tener cuidado! Su H , también conocido como operador de Laplace, es autoadjunto en L 2 ( R d ) .